[考研类试卷]考研数学三(线性方程组)模拟试卷4及答案与解析.doc

上传人:tireattitude366 文档编号:853168 上传时间:2019-02-22 格式:DOC 页数:15 大小:294KB
下载 相关 举报
[考研类试卷]考研数学三(线性方程组)模拟试卷4及答案与解析.doc_第1页
第1页 / 共15页
[考研类试卷]考研数学三(线性方程组)模拟试卷4及答案与解析.doc_第2页
第2页 / 共15页
[考研类试卷]考研数学三(线性方程组)模拟试卷4及答案与解析.doc_第3页
第3页 / 共15页
[考研类试卷]考研数学三(线性方程组)模拟试卷4及答案与解析.doc_第4页
第4页 / 共15页
[考研类试卷]考研数学三(线性方程组)模拟试卷4及答案与解析.doc_第5页
第5页 / 共15页
点击查看更多>>
资源描述

1、考研数学三(线性方程组)模拟试卷 4 及答案与解析一、选择题下列每题给出的四个选项中,只有一个选项符合题目要求。1 要使 1= 都是线性方程组 Ax=0 的解,只要系数矩阵 A 为2 设齐次线性方程组经高斯消元化成的阶梯形矩阵是 ,则自由变量不能取成(A)x 4,x 5(B) x2,x 3(C) x2,x 4 (D)x 1,x 33 设 A 是 mn 矩阵,则下列命题正确的是(A)如 mn,则 Ax=b 有无穷多解(B)如 Ax=0 只有零解,则 Ax=b 有唯一解(C)如 A 有 n 阶子式不为零,则 Ax=0 只有零解(D)Ax=b 有唯一解的充要条件是 r(A)=n4 非齐次线性方程组

2、Ax=b 中未知量的个数为 n,方程个数为 m,系数矩阵 A 的秩为 r,则正确命题是(A)r=m 时,方程组 Ax=b 有解(B) r=n 时,方程组 Ax=b 有唯一解(C) m=n 时,方程组 Ax=b 有唯一解(D)rn 时,方程组 Ax=b 有无穷多解5 已知 1, 2, 3, 4 是齐次方程组 Ax=0 的基础解系,则此方程组的基础解系还可以是(A) 1+2, 2+3, 3+4, 4+1 (B) 1, 2, 3+4, 3-4(C) 1, 2, 3, 4 的一个等价向量组 (D) 1, 2, 3, 4 的一个等秩的向量组6 设 A 是 54 矩阵,A=( 1, 2, 3, 4),若

3、1=(1,1,-2,1) T, 2=(0,1,0,1)T 是 Ax=0 的基础解系,则 A 的列向量组的极大线性无关组是(A) 1, 3(B) 2, 4 (C) 2, 3(D) 1, 2, 4二、填空题7 已知方程组 有无穷多解,则 a=_8 已知方程组 总有解,则 应满足_9 四元方程组 的基础解系是_10 四元方程组 Ax=b 的三个解是 1, 2, 3,其中 1=(1,1,1,1)T, 2+3=(2,3,4,5) T,如 r(A)=3,则方程组 Ax=b 的通解是_11 设 A 为三阶非零矩阵,B= ,且 AB=0,则 Ax=0 的通解是_12 设 A= A *是 A 的伴随矩阵,则 A

4、*x=0 的通解是_13 已知 1, 2, t 都是非齐次线性方程组 Ax=b 的解,如果 c11+c22+ctt仍是 Ax=b 的解,则 c1+c2+ct=_.14 已知方程组 的通解是(1,2,-1,0) T+k(-1,2,-1,1) T,则 a=_.15 已知 1=(-3,2,0) T, 2=(-1,0,-2) T 是方程组 的两个解,则此方程组的通解是_三、解答题解答应写出文字说明、证明过程或演算步骤。16 已知 A 是 mn 矩阵,其 m 个行向量是齐次线性方程组 Cx=0 的基础解系,B是 m 阶可逆矩阵,证明 BA 的行向量也是齐次方程组 Cx=0 的基础解系17 证明方程组 有

5、解的必要条件是行列式并举例说明该条件是不充分的18 已知方程组 有解,证明方程组无解19 设 A=(aij)是 mn 矩阵, =(b1,b 2,b n)是 n 维行向量,如果方程组()Ax=0的解全是方程()b 1x1+b2x2+bnxn=0 的解,证明 可用 A 的行向量1, 2, m 线性表出20 已知方程组 有解,证明:方程组的任意一组解必是方程()b1x1+b2x2+bmxm=0 的解21 求齐次方程组 的基础解系22 求线性方程组的所有解23 当 a,b 取何值时,方程细 有唯一解,无解,有无穷多解? 当方程组有解时,求其解24 设线性方程组 已知(1,-1,1,-1) T 是该方程

6、组的一个解,求方程组所有的解25 已知 a,b ,c 不全为零,证明方程组 只有零解26 设 A 是 n 阶矩阵,证明方程组 Ax=b 对任何 b 都有解的充分必要条件是A027 证明:与基础解系等价的线性无关的向量组也是基础解系考研数学三(线性方程组)模拟试卷 4 答案与解析一、选择题下列每题给出的四个选项中,只有一个选项符合题目要求。1 【正确答案】 A【试题解析】 由于 Ax=0 已有 2 个线性无关的解,故 n-r(A)2,即 r(A)1所以(B)、(D)的秩不符合题目要求 1 不是(C)中方程的解,因而 1 不是(C) 的解用排除法应选(A) 【知识模块】 线性方程组2 【正确答案】

7、 A【试题解析】 因为 =0,所以 x4,x 5 不能取为自由变量选(A)【知识模块】 线性方程组3 【正确答案】 C【试题解析】 如 mn,齐次方程组 Ax=0 有无穷多解,而线性方程组可以无解,两者不要混淆,请举简单反例 如 Ax=0 只有零解,则 r(A)=n,但由 r(A)=n 推断不出 =n,因此 Ax=b 可以无解例如 前者只有零解,而后者无解故(B)不正确采用排除法故(C)正确【知识模块】 线性方程组4 【正确答案】 A【试题解析】 A 是 mn 矩阵,r(A)=m 说明 A 的行向量组线性无关,那么增广矩阵 的行向量组是 A 的行向量组的延伸组,必线性无关故所以(A)正确【知识

8、模块】 线性方程组5 【正确答案】 B【试题解析】 向量组(A) 线性相关, (A)不正确 1, 2, 3, 4, 1+2 与1, 2, 3, 4 等价但前者线性相关,故(C)不正确等秩的向量组不一定能互相线性表出,因而可能不是方程组的解,故(D)不正确选 (B)【知识模块】 线性方程组6 【正确答案】 C【试题解析】 由 A1=0,知 1+2-23+4=0 由 A2=0,知 2+4=0 因为 n-r(A)=2,故必有 r(a)=2所以可排除(D) 由知, 2, 4 线性相关故应排除(B) 把代入得 1-23=0,即 1, 3 线性相关,排除 (A) 如果 2, 3 线性相关,则 r(1, 2

9、, 3, 4)=r(-23, 2, 3,- 2)=r(2, 3)=1 与 r(A)=2 相矛盾所以选(C) 【知识模块】 线性方程组二、填空题7 【正确答案】 -5【试题解析】 对增广矩阵作初等行变换,有当 a=-5 时,r(A)= ,方程组有无穷多解【知识模块】 线性方程组8 【正确答案】 【试题解析】 对任意 b 1, b2,b 3,方程组有解 A0而由【知识模块】 线性方程组9 【正确答案】 (0,0,1,0) T,(-1,1,0,1) T【试题解析】 n-r(A)=4-2=2 取 x3,x 4 为自由变量: 令 x3=1,x 4=0 得x2=0, x1=0;令 x3=0,x 4=1 得

10、 x2=1,x 1=-1, 所以基础解系是(0,0,1,0) T,(-1,1,0,1) T【知识模块】 线性方程组10 【正确答案】 (1,1,1,1) T+k(0,1,2,3) T【试题解析】 由( 2+3)-21=(2-1)+(3-1)=(2,3 ,4,5) T-2(1,1,1,1)T=(0,1,2,3) T,知(0,1,2,3) T 是 Ax=0 的解 又秩 r(A)=3,n-r(A)=1,所以Ax=b 的通解是(1,1,1, 1)T+k(0,1,2,3) T【知识模块】 线性方程组11 【正确答案】 k 1(1,4,3) T+k2(-2,3,1) T【试题解析】 因为 AB=0,A0,

11、所以 r(A)+r(B)3,r(A)1故 r(B)2又因 B中有 2 阶子式不为 0,所以秩 r(B)2从而 r(B)=2故 r(A)=1于是 n-r(A)=2 由 AB=0 又知 B 的列向量是齐次方程组的解,所以 Ax=0 的通解是 k1(1,4,3)T+k2(-2,3,1) T【知识模块】 线性方程组12 【正确答案】 k 1(1,4,7) T+k2(2,5,8) T【试题解析】 因为秩 r(A)=2,所以行列式A=0 那么 A*A=AE=0,所以A 的列向量是 A*x=0 的解 又因 r(A*)=1,故 A*x=0 的通解是 k1(1,4,7)T+k2(2,5,8) T【知识模块】 线

12、性方程组13 【正确答案】 1【试题解析】 因为 i 是 Ax=b 的解,所以,A i=b 若 c11+c22+ctt 是 Ax=b的解,则 A(c 11+c22+ctt)=c1A1+c2A2+ctAt =(c1+c2+ct)b=b 故c1+c2+ct=1【知识模块】 线性方程组14 【正确答案】 3【试题解析】 因(1,2,-1,0) T 是 Ax=b 的解,则将其代入第 2 个方程可求出b=1 因(-1,2,-1,1) T 是 Ax=0 的解,则将其代入第 1 个方程可求出 a=3【知识模块】 线性方程组15 【正确答案】 (-3,2,0) T+k(-1,1,1) T【试题解析】 由于矩阵

13、 A 中有 2 阶子式不为 0,故秩 r(A)2 又 1-2 是 Ax=0 的非零解,知 r(A)3 故必有 r(A)=2于是 n-r(a)=1 所以方程组通解是:(-3,2,0) T+k(-1,1,1) T【知识模块】 线性方程组三、解答题解答应写出文字说明、证明过程或演算步骤。16 【正确答案】 因为 A 的行向量是 Cx=0 的解,即 CAT=0,那么 C(BA)T=CATBT=OBT=0 可见 BA 的行向量是方程组 Cx=0 的解 由于 A 的行向量是基础解系,所以 A 的行向量线性无关,于是 m=r(A)=n-r(C) 又因 B 是可逆矩阵,r(BA)=r(A)=m=n-r(C),

14、所以 BA 的行向量线性无关,其向量个数正好是 n-r(C),从而是方程组 Cx=0 的基础解系【知识模块】 线性方程组17 【正确答案】 如果方程组 Ax=b 有解,则 r(A)= ;因为 A 是(n+1)n 阶矩阵,必有 r(A)n,所以 n那么,必有 n+1 阶行列式例如,方程组=0,但该方程组无解【知识模块】 线性方程组18 【正确答案】 用 分别表示方程组()与()的系数矩阵和增广矩阵,易见 A2= 因为方程组( )有解,故 r(A1)= 又由于(b1,b 2,n m,1) 不能由(a 11,a 21,a m1,0),(a 12,a 22,a m2,0),(a 1n,a 2n,a m

15、n, 0)线性表出,所以【知识模块】 线性方程组19 【正确答案】 构造一个联立方程组 简记为 Cx=0,显然, ()的解必是()的解,又因()的解全是()的解,于是( )的解也必全是() 的解,所以() ,()是同解方程组,它们有相同的解空间从而n-r(A)=n-r(C),即 r(A)=r(C),亦即 r(1, 2, m)=r(1, 2, m,) 因此极大线性无关组所含向量个数相等,这样 1, 2, m 的极大线性无关组也必是 1, , m, 的极大线性无关组,从而 可由 1, 2, m 线性表出【知识模块】 线性方程组20 【正确答案】 1 记方程组 ()的系数矩阵为 A,增广矩阵是 ,由

16、于()有解,故 r(A)= 那么(b 1,b 2,b m)T 可用 A 的列向量线性表出联立()、(),得方程组 显然,系数矩阵是,可见方程组()中最后一个方程是多余的,即() 与 ()是同解方程组,这就是 ()的任一解必是( )的解2记y=(y1,y 2,y n)T,x=(x 1,x 2,x m)T,b=(b 1,b 2,b m)T 由于()有解,故存在 y 使 Ay=b,那么 bT=yTAT设 x 是方程组()A Tx=0 的任一解,于是bTx=yTATx=yT0=0,即 b1x1+b2x2+bmxm=0,即()的解必是()的解【知识模块】 线性方程组21 【正确答案】 对系数矩阵作初等变

17、换,有当 a1 时,r(A)=3 ,取自由变量 x4 得 x4=1,x 3=0,x 2=-6,x 1=5基础解系是(5,-6,0,1) T 当 a=1 时,r(A)=2取自由变量 x3,x 3,则由 x 3=1,x 4=0 得 x2=-2,x 1=1, x 3=0,x 4=1 得 x2=-6,x 1=5,知基础解系是 (1,-2 ,1,0) T,(5,-6 ,0,1) T【知识模块】 线性方程组22 【正确答案】 对增广矩阵作初等行变换,有方程组的解:令 x3=0, x4=0 得 x2=1,x 1=2即 =(2,1,0,0) T导出组的解: 令x3=1, x4=0 得 x2=3,x 1=1即

18、1=(1,3,1,0) T; 令 x3=0,x 4=1 得 x2=0,x 1=-1即 2=(-1,0,0,1) T 因此方程组的通解是:(2,1,0,0) T+k1(1,3,1,0)T+k2(-1,0,0,1) T 而其中满足 的解,即(2+k 1-k2)2=(1+3k1)2那么 2+k 1-k2=1+3k1 或 2+k1-k2=-(1+3k1),即 k 2=1-2k1 或 k2=3+4k1 所以(1,1,0,1)T+k(3,3,1,-2) T 或(-1 ,1,0,3) T+k(-3,3,1,4) T 为满足 的所有解【知识模块】 线性方程组23 【正确答案】 对增广矩阵作初等行变换,有()当

19、 a0,且 b3时,方程组有唯一解 ()当 a=0 时, 方程组均无解()当 a0,b=3时,方程组有无穷多解【知识模块】 线性方程组24 【正确答案】 将(1,-1,1,-1) T 代人方程组得 =对增广矩阵作初等行变换,有因 r(A)= =34,所以方程组有无穷多解 (-1,0,0,1) T+k(2,-1,1,-2) T【知识模块】 线性方程组25 【正确答案】 因为系数行列式所以齐次方程组只有零解【知识模块】 线性方程组26 【正确答案】 必要性对矩阵 A 按列分块 A=(1, 2, n),则 1, 2, n 可表示任何 n 维向量 b 1, 2, n 可表示e1=(1,0,0,0) T

20、,e 2=(0,1,0,0) T, ,e n=(0,0,0,1) T r(1, 2, n)r(e1,e 2,e n)=n r(A)=n所以A 0充分性由克莱姆法则,行列式A0 时方程组必有唯一解,故 ,Ax=b 总有解【知识模块】 线性方程组27 【正确答案】 设 Ax=0 的基础解系是 1, 2, t若 1, 2, s 线性无关, 1, 2, s 与 1, 2, t 等价 由于 j(j=1,2,s)可以由1, 2, t 线性表示,而 i(i=1,t)是 Ax=0 的解,所以 j (j=1,2,s)是 Ax=0 的解 因为 1, 2, t 线性无关,秩 r(1, 2, t)=t,又1, 2, t 与 1, 2, s 等价,所以 r(1, 2, s)=r(1, 2, t)=t又因 1, 2, s 线性无关,故 s=t. 因此 1, 2, t 是 Ax=0 的基础解系【知识模块】 线性方程组

展开阅读全文
相关资源
猜你喜欢
相关搜索

当前位置:首页 > 考试资料 > 大学考试

copyright@ 2008-2019 麦多课文库(www.mydoc123.com)网站版权所有
备案/许可证编号:苏ICP备17064731号-1